If you find any mistakes, please make a comment! Thank you.

Solution to Principles of Mathematical Analysis Chapter 9 Part A


Chapter 9 Functions of Several Variables

Exercise 1

(By analambanomenos) Let $\mathbf x,\mathbf y$ be in the span of $S$ and let $c$ be a scalar; we need to show that $\mathbf x+\mathbf y$ and $c\mathbf x$ are in the span of $S$. We have
\begin{align*}
\mathbf x &= c_1\mathbf x_1+\cdots+c_m\mathbf x_m \\
\mathbf y &= d_1\mathbf y_1+\cdots+d_n\mathbf y_n
\end{align*} for some $\mathbf x_1,\ldots,\mathbf x_m,\mathbf y_1,\ldots,\mathbf y_n$ in $S$ and some scalars $c_1,\ldots,c_m,d_1,\ldots,d_n$. Hence
\begin{align*}
\mathbf x+\mathbf y &= c_1\mathbf x_1+\cdots+c_m\mathbf x_m+d_1\mathbf y_1+\cdots+d_n\mathbf y_n \\
c\mathbf x &= cc_1\mathbf x_1+\cdots+cc_m\mathbf x_m
\end{align*}shows that $\mathbf x+\mathbf y$ and $c\mathbf x$ are in the span of $S$.


Exercise 2

(By analambanomenos) Let $A\in L(X,Y)$ and $B\in L(Y,Z)$. Let $\mathbf x,\mathbf y\in X$ and let $c$ be a scalar. Then
\begin{align*}
(BA)(\mathbf x+\mathbf y) &= B\big(A(\mathbf x+\mathbf y)\big) \\
&= B\big(A(\mathbf x)+A(\mathbf y)\big) \\
&= B\big(A(\mathbf x)\big) + B\big(A(\mathbf y)\big) \\
&= (BA)(\mathbf x)+(BA)(\mathbf y) \\
(BA)(c\mathbf x) &= B\big(A(c\mathbf x)\big) \\
&= B\big(cA(\mathbf x)\big) \\
&= cB\big(A(\mathbf x)\big) \\
&= c(BA)(\mathbf x)
\end{align*}which shows that $BA$ is linear.

Let $A\in L(X)$ be invertible. Let $\mathbf x,\mathbf y\in X$ and let $c$ be a scalar. Then there are $\mathbf x’,\mathbf y’\in X$ such that
\[
A(\mathbf x’)=\mathbf x, \quad A(\mathbf y’)=\mathbf y, \quad A^{-1}(\mathbf x)=\mathbf x’, \quad A^{-1}(\mathbf y)=\mathbf y’
\]Then
\begin{align*}
A^{-1}(\mathbf x+\mathbf y) &= A^{-1}\big(A(\mathbf x’)+A(\mathbf y’)\big) \\
&= A^{-1}\big(A(\mathbf x’+\mathbf y’)\big) \\
&= \mathbf x’+\mathbf y’ \\
&= A^{-1}(\mathbf x)+A^{-1}(\mathbf y) \\
A^{-1}(c\mathbf x) &= A^{-1}\big(cA(\mathbf x’)\big) \\
&= A^{-1}\big(A(c\mathbf x’)\big) \\
&= c\mathbf x’ \\
&= cA^{-1}(\mathbf x)
\end{align*}which shows that $A^{-1}$ is linear. Since $A^{-1}$ is 1-1 and maps $X$ onto itself, it is also invertible.


Exercise 3

(By analambanomenos) Suppose $A(\mathbf x_1)=A(\mathbf x_2)$. Then \[
A(\mathbf x_1-\mathbf x_2)=A(\mathbf x_1)-A(\mathbf x_2)=\mathbf 0 \text{ implies that }\mathbf x_1-\mathbf x_2=\mathbf 0 \] so that $\mathbf x_1=\mathbf x_2$. Hence $A$ is 1-1.


Exercise 4

(By analambanomenos) Let $\mathbf x,\mathbf y\in\mathscr N(A)$ and let $c$ be a scalar. Then
\begin{align*}
A(\mathbf x+\mathbf y) &= A(\mathbf x)+A(\mathbf y) = \mathbf 0+\mathbf 0 = \mathbf 0 \text{ so that }\mathbf x+\mathbf y\in\mathscr N(A) \\
A(c\mathbf x) &= cA(\mathbf x) = c\mathbf 0 = \mathbf 0 \text{ so that }c\mathbf x\in\mathscr N(A)
\end{align*} which shows that $\mathscr N(A)$ is a vector space.

Let $\mathbf x,\mathbf y\in\mathscr R(A)$, so that $\mathbf x=A(\mathbf x’)$ and $\mathbf y=A(\mathbf y’)$, and let $c$ be a scalar. Then
\begin{align*}
\mathbf x+\mathbf y &= A(\mathbf x’)+A(\mathbf y’) = A(\mathbf x’+\mathbf y’) \text{ so that }\mathbf x+\mathbf y\in\mathscr R(A) \\
c\mathbf x &= cA(\mathbf x’) = A(c\mathbf x’) \text{ so that }c\mathbf x\in\mathscr R(A)
\end{align*}which shows that $\mathscr R(A)$ is a vector space.


Exercise 5

(By analambanomenos) I’m going to show this by induction on $n$. For the case $n=1$, let $A\mathbf e_1=k$, and let $\mathbf y=k\mathbf e_1$. If $\mathbf x\in \R1$, then $\mathbf x=x\mathbf e_1$ for some scalar $x$, so $A\mathbf x=xA\mathbf e_1=ck=\mathbf x\cdot\mathbf y$.

Suppose the assertion is true for the case $n$ and let $A\in L(\R{n+1},\R1)$. Restricting $A$ to the subspace $\R n$ (spanned by $\mathbf e_1,\ldots,\mathbf e_n$) yields an element of $L(\R n,\R1)$, so by the induction assumption there is a $\mathbf y’\in\R n$ such that $A\mathbf x’=\mathbf x’\cdot\mathbf y’$ for all $\mathbf x’\in\R n$. If $A\mathbf e_{n_1}=k$, let $\mathbf y=\mathbf y’+k\mathbf e_{n+1}$. If $\mathbf x\in\R{n+1}$, then $\mathbf x=\mathbf x’+x\mathbf e_{n+1}$ for some $\mathbf x’\in\R n$ and some scalar $x$. Then, since the scalar product of $\mathbf e_{n+1}$ with every element of $\R n$ is 0, we have
\begin{align*}
A\mathbf x &= A\mathbf x’+xA\mathbf e_{n+1} \\
&= \mathbf x’\cdot \mathbf y’ + xk \\
&= (\mathbf x’+x\mathbf e_{n+1})\cdot\mathbf y’ + (\mathbf x’+x\mathbf e_{n+1})\cdot (k\mathbf e_{n+1}) \\
&= \mathbf x\cdot\mathbf y
\end{align*}If $|{\mathbf x}|\le1$, then $|{A\mathbf x}|=|{\mathbf x\cdot\mathbf y}|\le|{\mathbf x}|\,|{\mathbf y}|\le|{\mathbf y}|$, so that $\|{A}\|\le|{\mathbf y}|$. Let $\tilde{\mathbf y}=\mathbf y/|{\mathbf y}|$. Then $|{\tilde{\mathbf y}}|=1$ and
\[|{A\tilde{\mathbf y}}|=\frac{|{\mathbf y\cdot\mathbf y}|}{|{\mathbf y}|}=\frac{|{\mathbf y}|^2}{|{\mathbf y}|}=|{\mathbf y}|.
\]Hence $\|{A}\|=|{\mathbf y}|$.


Exercise 6

(By analambanomenos) The usual rules of differentiaion show that the partial derivatives of $\R2$ at $(x,y)\ne(0,0)$ are\[D_1f(x,y)=\frac{y(y^2-x^2)}{(x^2+y^2)^2} \qquad D_2f(x,y)=\frac{x(x^2-y^2)}{(x^2+y^2)^2}\]And since $f$ is equal to 0 everywhere along the $x$ and $y$ axes, they also exist and are equal to 0 at $(0,0)$.

For $x\ne 0$, $f(x,x)=x^2/(x^2+x^2)=1/2$. Since $f(0,0)=0$, $f$ is not continuous along the line $y=x$ at $(0,0)$.


Exercise 7

(By analambanomenos) Suppose that $|D_jf|<M$ in $E$, for $j=1,\ldots,n$. Following the hint to mimic the proof of Theorem 8.21, fix $\mathbf x\in E$ and let $\varepsilon>0$. Since $E$ is open, there is an open ball $S\subset E$, with center at $\mathbf x$ and radius $r<(Mn)^{-1}$. Suppose $\mathbf h=\sum h_j\mathbf e_j$, $|\mathbf h|<r$, put $\mathbf v_0=\mathbf 0$, and $\mathbf v_k=h_1\mathbf e_1+\cdots+h_k\mathbf e_k$, for $1\le k\le n$. Then
\begin{equation}\label{9.7.1}f(\mathbf x+\mathbf h)-f(\mathbf x)=\sum_{j=1}^n\big(f(\mathbf x+\mathbf v_j)-f(\mathbf x+\mathbf v_{j-1})\big). \end{equation}Since $| \mathbf v_k|<r$ for $1\le k\le n$ and since $S$ is convex, the segments with endpoints $\mathbf x+\mathbf v_{j-1}$ and $\mathbf x+\mathbf v_j$ lie in $S$. Since $\mathbf v_j=\mathbf v_{j-1}+h_j\mathbf e_j$, the mean value theorem, Theorem 5.10, shows that the $j$th summand in \eqref{9.7.1} is equal to
\[
h_j(D_jf)(\mathbf x+\mathbf v_{j-1}+\theta_jh_j\mathbf e_j)
\] for some $\theta_j\in(0,1)$. By \eqref{9.7.1}, it follows that
\begin{align*}
\big|f(\mathbf x+\mathbf h)-f(\mathbf x)\big| &\le \sum_{j=1}^n\big|f(\mathbf x+\mathbf v_j)-f(\mathbf x+\mathbf v_{j-1})\big| \\
&= \sum_{j=1}^n\big|h_j(D_jf)(\mathbf x+\mathbf v_{j-1}+\theta_jh_j\mathbf e_j)\big| \\
&< \sum_{j=1}^n|h_j|M<(Mn)r<\varepsilon
\end{align*}which shows that $f$ is continuous at $\mathbf x$. Since $\mathbf x\in E$ was arbitrary, we have $f$ continuous on $E$.


Exercise 8

(By analambanomenos) You can use Theorem 9.17 to express $f’$ as a sum of the partial derivatives and easily reduce the problem to the the single-variable case, Theorem 5.8. However, I thought I’d use the new definition of derivative (commonly called a Fr\’echet derivative, by the way) instead.

By Exercise 5, there is a $\mathbf y\in\R n$ such that $f’(\mathbf x)=\mathbf y\cdot\mathbf x$. Let $\mathbf h=h\mathbf y/|\mathbf y|$, and take the limit in the definition of derivative as $h$ approaches 0 through positive numbers. Then we get
\begin{align*}
0 &= \lim_{\mathbf h\rightarrow \mathbf 0}\frac{\big|f(\mathbf x+\mathbf h)-f(\mathbf x)-f’(\mathbf x)\mathbf h\big|}{|\mathbf h|} \\
&= \lim_{h\rightarrow 0+}\frac{\big|f(\mathbf x+\mathbf h)-f(\mathbf x)-\mathbf y\cdot(h\mathbf y)/|\mathbf y|\big|}{h|\mathbf y|/|\mathbf y|} \\
&= \lim_{h\rightarrow 0+}\frac{\big|f(\mathbf x+\mathbf h)-f(\mathbf x)-h|\mathbf y|\big|}{h} & \hbox{(since $\mathbf y\cdot\mathbf y=|\mathbf y|^2$)} \\
&= \lim_{h\rightarrow 0+}\frac{f(\mathbf x)-f(\mathbf x+\mathbf h)+h|\mathbf y|}{h} & \hbox{(since $f(\mathbf x+\mathbf h)-f(\mathbf x)$ and $-h|\mathbf y|$ are $\le0$)} \\
&= |\mathbf y| + \lim_{h\rightarrow 0+}\frac{f(\mathbf x)-f(\mathbf x+\mathbf h)}{h}
\end{align*}Since the term in the limit is non-negative for small $h$, this forces $|\mathbf y|\le 0$, or $\mathbf y=\mathbf 0$. Hence $f’(\mathbf x)=0$.


Exercise 9

(By analambanomenos) Fix $\mathbf x\in E$. Since $E$ is open, there is a open ball $S\subset E$ containing $\mathbf x$. By the Corollary to Theorem 9.19, $\mathbf f$ is constant on $S$. Hence the set $E’$ of all points $\mathbf z\in E$ such that $\mathbf f(\mathbf z)=\mathbf f(\mathbf x)$ is an open subset of $E$. Similarly, the set $E-E’$ is also open in $E$ (being the union of open sets on which $\mathbf f$ has a constant value not equal to $\mathbf f(\mathbf x)$). By Exercise 2.19(b), $E’$ and $E-E’$ are two separated sets whose union is $E$. Since $E$ is connected, we must have $E’=E$, so $\mathbf f$ is constant on $E$.


Exercise 10

(By analambanomenos) Let $E$ satisfy the weaker condition: if $(x,x_2,\ldots,x_n)\in E$ and $(y,x_2,\ldots,x_n)\in E$, then for all $x<z<y$ we have $(z,x_2,\ldots,x_n)\in E$. For such points, define $f_{x_2,\ldots,x_n}(z)=f(z,x_2,\ldots,x_n)$, then $f_{x_2,\ldots,x_n}’(z)=(D_1f)(z,x_2,\ldots,x_n)=0$, so by the Mean Value Theorem $f_{x_2,\ldots,x_n}$ must be constant on $[x,y]$. Hence $f(z,x_2,\ldots,x_n)$ must equal this constant value for all $z$ such that $(z,x_2,\ldots,x_n)\in E$, so that $f(\mathbf x)$ depends only on $x_2,\ldots,x_n$.

To see a counterexample, let $E\subset\R2$ be the square $(x,y)$, $-1<x<1$, $-1<y<1$ less the positive $y$-axis, $(0,y)$, $0<y<1$. Define
\[
f(x,y)=
\begin{cases}
0 & -1<x<1,\;-1<y<0 \\
-y & -1<x<0,\;0\le y<1 \\
y & 0<x<1,\;0\le y<1
\end{cases}
\]Then $f$ is continuous, $(D_1f)(x,y)=0$ for all $(x,y)\in E$, but the value of $f$ does not depend only on $y$.


Exercise 11

(By analambanomenos) We have \begin{align*}
\nabla(fg) &= \sum_{i=1}^n\big(D_i(fg)\big)\mathbf e_i \\
&= \sum_{i=1}^n\big(f(D_ig)+g(D_if)\big)\mathbf e_i \\
&= f\sum_{i=1}^n(D_ig)\mathbf e_i + g\sum_{i=1}^n(D_if)\mathbf e_i \\
&= f\nabla g+g\nabla f \\[3ex]
0 &= \nabla(1) \\
&= \nabla(f\cdot f^{-1}) \\
&= f\nabla(f^{-1}) + f^{-1}\nabla f \\
f\nabla(f^{-1}) &= -f^{-1}\nabla f \\
\nabla(f^{-1}) &= -f^{-2}\nabla f
\end{align*}


Exercise 12

(By analambanomenos) The range of $\mathbf f$ is a torus with inner radius $b-a$ and outer radius $b+a$, centered at the origin, whose plane is perpendicular to the $z$-axis. To see this, the central circle of such a torus is the set of points $(b\cos t,b\sin t,0)$. Then the circle of radius $a$ which intersects the half-plane that goes through the $z$-axis and the point $(b\cos t,b\sin t,0)$ is the circle centered at that point and perpendicular to the $x$-$y$-plane, with radius vector $a(\cos t,\sin t,0)$, which is described by \[(b\cos t,b\sin t,0)+a(\cos t,\sin t, 0)\cos s+ a(0,0,1)\sin s = \big(f_1(s,t),f_2(s,t),f_3(s,t)\big).\]Note that $\mathbf f$ is a one-to-one mapping of the square $S$ given by $(x,y)$, $0\le x<2\pi$, $0\le y<2\pi$ onto the torus.

(a) Taking the partial derivatives of $f_1$ with respect to $s$ and $t$ to get $\nabla f_1$, we want to solve\[(\nabla f_1)(s,t)=\big(-a\sin s\cos t,-(b+a\cos s)\sin t\big)=(0,0).\]Restricing ourselves to the square $S$, the first component equals 0 when either $s=0$, $s=\pi$, $t=\pi/2$, or $t=3\pi/2$. Since $b+a\cos s>0$ for all $s$, the second component equals 0 only when $t=0$ or $t=\pi$. Hence $\nabla f_1$ equals $\mathbf 0$ in $S$ only at the four points\[
(0,0),\quad(0,\pi),\quad(\pi,0),\quad(\pi,\pi),
\]which map onto the four points\[
(b+a,0,0),\quad(-b-a,0,0),\quad(b-a,0,0),\quad(a-b,0,0),
\]respectively.

(b) Taking the partial derivatives of $f_3$ with respect to $s$ and $t$ to get $\nabla f_3$, we want to solve\[
(\nabla f_3)(s,t)=(a\cos s,0)=(0,0).
\]which occurs in $S$ at the points $(s,t)$, where $s=\pi/2$ or $s=3\pi/2$, and $0\le t<2\pi$. The points $(\pi/2,t)$, $0\le t<2\pi$, are mapped by $\mathbf f$ to the circle $(b\cos t, b\sin t, a)$, and the points $(3\pi/2,t)$, $0\le t<2\pi$, are mapped to the circle $(b\cos t, b\sin t, -a)$.

(c) At $(b+a,0,0)$, corresponding to $(s,t)=(0,0)$, where $\cos s=\cos t=1$, $f_1$ attains its maximum value, $b+a$, and at $(-b-a,0,0)$, corresponding to $(s,t)=(0,\pi)$, where $\cos s=1$ and $\cos t=-1$, $f_1$ attains its minimum value, $-b-a$. At $(b-a,0,0)$, corresponding to $(s,t)=(\pi,0)$, where $\cos s=-1$ and $\cos t=1$, $f_1$ increases if you fix $t=0$ and vary $s$, and decreases if you fix $s=\pi$ and vary $t$. Similarly, at $(a-b,0,0$, corresponding to $(s,t)=(0,\pi)$, where $\cos s=1$ and $\cos t=-1$, $f_1$ decreases if you fix $t=\pi$ and vary $s$, and increases if you fix $s=\pi$ and vary $t$.

The points $(b\cos t,b\sin t,a)$, $0\le t<2\pi$, correspond to the points $(s,t)=(\pi/2,t)$ where $f_3$ attains its maximum value of $a$, and $(b\cos t,b\sin t,-a)$, $0\le t<2\pi$, correspond to the points $(s,t)=(3\pi/2,t)$ where $f_3$ attains its minimum value of $-a$.

(d) To show that the “irrational winding of the torus” (it even has its own Wikipedia page) is dense, I first show a general proposition about continuous functions. Suppose $F$ is a continuous mapping from $X$ to $Y$ (both metric, or even just topological spaces). Then if $D$ is a dense subset of $X$, then $F(D)$ is a dense subset of $F(Y)$. For if not, then there would be a nonempty open set $G$ in $F(Y)$ disjoint from $F(D)$. Then $F^{-1}(D)$ would be an open subset of $X$ disjoint from $D$, contradicting the density of $D$.

Hence, to show that the image of $\mathbf g$ is dense in the torus $K$, it suffices to show that its preimage under the mapping $\mathbf f$ is dense in $\R2$. Call this set $K’$, it is the set of points of the form\[
(s+m2\pi,\lambda s+n2\pi),\qquad\hbox{$s$ real, $m$ and $n$ integers}.
\]First I want to show that the intersection of $K’$ with the $s$-axis is dense on the axis. If $(s,0)$ is a point in this set, then from the above, we that $\lambda s$ is a multiple of $2\pi$, so $s=(n\lambda^{-1}+m)2\pi$ for some integers $m$ and $n$. To see that this set is dense, I first show another general proposition.

If $\alpha$ is an irrational number, then the set of points $D$ in $[0,1]$ which are equivalent to integral multiples of $\alpha$ modulo 1 are dense in that interval. That is, let $n$ be an integer, then $n\alpha$ is in the interval $(m,m+1)$ for some integer $m$, so that $x_n=n\alpha-m$ is between 0 and 1. These points are all distinct, since if $x_{n_1}=n_1\alpha-m_1=x_{n_2}=n_2\alpha-m_2$, for some integers $n_1\ne n_2$, then $\alpha=(m_1-m_2)/(n_2-n_1)$ would be rational. Hence $D$ is an infinite set.

Another simple fact is that if $x_{n_1}<x_{n_2}$ are two points in $D$ then $0<x_{n_2}-x_{n_1}<1$, so that $x_{n_2}-x_{n_1} = (n_2-n_1)\alpha-(m_2-m_1)$ is also in $D$. Also, if $0<jx_n<1$ for some positive integer $j$, then $jx_{n}=jn\alpha-jm$ is also in $D$.

Let $\varepsilon>0$ and let $k$ be an integer large enough so that $k^{-1}<\varepsilon$. Divide $[0,1]$ into $k$ intervals \[
I_j=\big[jk^{-1},(j+1)k^{-1}\big]\qquad j=0,\ldots,(k-1)
\](This isn’t strictly a partition, but I am ignoring the points on the boundaries, which are rational numbers which cannot be in $D$.) Since $D$ is infinite, one of these intervals contains at least two points in $D$, say $x_{n_1}<x_{n_2}$. Then $x_{n_2}-x_{n_1}$ is a point $x_n\in D$ such that $0<x_n<k^{-1}$. Hence each of the intervals $I_j$, $j=0,\ldots,(k-1)$, contains a point $jx_n$ of $D$. If $x\in[0,1]$, then $x$ is in one of the $I_j$, so there is a point $x_n\in D$ such that $|x-x_n|<k^{-1}<\varepsilon$. Hence $D$ is dense in $[0,1]$.

Going back to the intersection of $K’$ with the $s$-axis, it is not hard to see that the intersection of $K’$ with the interval $[0,2\pi]$ is the image of the set $D$ above, with $\alpha=\lambda^{-1}$, under the mapping $s\mapsto 2\pi s$, hence $K’$ is dense in the interval $[0,2\pi]$ in the $s$-axis. Since the other points of $K’$ on the $s$-axis are translates of this dense set by multiples of $2\pi$, we see that $K’$ is dense in the entire axis.

For other lines in $\R2$ parallel to the $s$-axis, $(s,t)\in K’$ if $(s-\lambda^{-1}t,0)\in K’$, that is, the intersection of $K’$ with this line is the image of the dense interesection of $K’$ with the $s$-axis under the translation $(s,0)\mapsto(s+\lambda^{-1}t,t)$. Hence $K’$ is also dense in this line, and so is dense in all of $\R2$.

If $\mathbf g(t_1)=\mathbf g(t_2)$, then $\mathbf f(t_1,\lambda t_1)=\mathbf f(t_2,\lambda t_2)$ which would only happen if $(t_1-t_2)=n2\pi$ and $\lambda(t_1-t_2)=m2\pi$ for some integers $m$ and $n$. But that would imply $\lambda=m/n$ which contradicts the irrationality of $\lambda$.

By Theorem 9.17 we have
\begin{align*}
\mathbf f’(s,t)\mathbf e_1 &= \big((D_1f_1)(s,t),(D_1f_2)(s,t),(D_1f_3)(s,t)\big) \\
&= \big(-a\sin s\cos t,-a\sin s\sin t,a\cos s\big) \\
\mathbf f’(s,t)\mathbf e_2 &= \big((D_2f_1)(s,t),(D_2f_2)(s,t),(D_2f_3)(s,t)\big) \\
&= \big(-(b+a\cos s)\sin t,(b+a\cos s)\cos t,0\big).
\end{align*} If we let ${\boldsymbol\gamma}(t)=(t,\lambda t)$, so that $\boldsymbol\gamma’(t)=\mathbf e_1+\lambda\mathbf e_2$, then we have by the chain rule
\begin{align*}
\mathbf g’(t) &= \mathbf f’\big(\boldsymbol\gamma(t)\big)\boldsymbol\gamma’(t) \\
&= \mathbf f’(t,\lambda t)(\mathbf e_1+\lambda\mathbf e_2) \\
&= \big(-a\sin t\cos\lambda-\lambda(b+a\cos t)\sin\lambda,-a\sin t\sin\lambda t+\lambda(b+a\cos t)\cos\lambda t,a\cos t\big) \\
\big|\mathbf g’(t)\big|^2 &= a^2\sin^2t\cos^2\lambda t+2\lambda a(b+a\cos t)\sin t\cos\lambda t\sin\lambda t+\lambda^2(b+a\cos t)^2\sin^2\lambda t\;+ \\
&\phantom{=}\;\; a^2\sin^2t\sin^2\lambda t-2\lambda a(b+a\cos t)\sin t\sin\lambda t\cos\lambda t+\lambda^2(b+a\cos t)^2\cos^2\lambda t\;+ \\
&\phantom{=}\;\; a^2\cos^2t \\
&= a^2+\lambda^2(b+a\cos t)^2.
\end{align*}

Baby Rudin 数学分析原理完整第九章习题解答

Linearity

This website is supposed to help you study Linear Algebras. Please only read these solutions after thinking about the problems carefully. Do not just copy these solutions.
Close Menu